Turn 2 1/8 into an improper fraction. Explain your steps

Answers

Answer 1

Answer:

17/8

Step-by-step explanation:

2 1/8    multiply 2 times 8 and 1 which equals to 17/8. The whole number times the denominator plus the numerator basically.

Answer 2

Answer:

17/8

Step-by-step explanation:

8 x 2 = 16

16 + 1 = 17


Related Questions

A=1/2h(a+b), solve for a

Answers

Step-by-step explanation:

2A=h(a+b)

2A=ha+hb

2A-hb=ha

2A-hb/h=a

Multiply by 2 on both sides to get:
2A = h(a + b)

Divide by h on both sides to get:
2A/h = a + b

Subtract b from both sides to get:
(2A/h) - b = a

Find the 15th term of the arithmetic sequence if the first term is -3 and the common difference is 2. Write the equation first, then substitute to find the answer

Answers

Answer:

25.

Step-by-step explanation:

f(x)=-3+2(x-1)

f(15)=-3+2(15-1)

f(15)=-3+2(14)

f(15)=-3+28

f(15)=25

Describe how to write a three-digit number in three different ways. ​

Answers

Answer:

Step-by-step explanation:

how do u want me to solve it

Over the past week, 40% of the phone calls to Alicia's cell phone were from her son. Let's say you know that her son called her 12 times. Find out how many total calls she received from her son.

Answers

Answer:

40%

Step-by-step explanation: hope this helps

Answer:

She received 12 calls from her son and 30 calls in total.

Step-by-step explanation:

40% of 30 is 12.

what is the answer for this -6+x=-11

Answers

Answer:

x = -5

Step-by-step explanation:

-6 + x = -11

x = -11 + 6

x = -5

Answer:

x=17

Step-by-step explanation:

17+ -6= 11

pls mark brainlest for real ('>')

(50 points!)
Find the missing angles​

Answers

Answer:

<2= 133

<3 = 47

<4 = 133

<5 = 47

<6 = 133

<7 = 47

<8 = 133

Step-by-step explanation:

im assuming a transversal is cutting the two lines, and that the 2 lines are parallel

2 = u must do 180-47 since theyre supplementary angles

3 = they are vertical angles

4 = 4 is a vertical angle to 2, but you can also do 180-47 since it is supp. to 3

5 = corresponding angle to the 47 degrees one

6 = alternate interior with 4, but also corresponding with 2

7 = vertical angles with 5 but also alternate exterior with the 47 degree angle , but also supplementary with both 6 and 8, and corresponding with 3

8= corresponding with 4, but also supplementary to 7 and 5. and vertical to 6

Answer:

<2= 133  <3 = 47   <4 = 133  <5 = 47  <6 = 133  <7 = 47  <8 = 133

Write 257% as decimal

Answers

Answer:

please mark brainliest!

Step-by-step explanation:

2.57

Answer:

2.57

Hope this helps! Mark me brainliest

god bless!

what’s 3/4 of 2/5 ?

** I WILL MAKE UP BRAIN LIST OR SOMETHING

Answers

Answer:

[tex] \frac{3}{10} [/tex]

Step-by-step explanation:

[tex] = \frac{3}{4} \times \frac{2}{5} [/tex]

[tex] = \frac{3 \times 2}{4 \times 5} [/tex]

[tex] = \frac{6}{20} [/tex]

[tex] = \frac{6 \div 2}{20 \div 2} [/tex]

[tex] = \frac{3}{10} [/tex]

Answer:

[tex] \frac{3}{10} [/tex]

Step-by-step explanation:

This is a shorter explanation. It's better to use this if you are good at maths or in a higher level.

[tex] \frac{3}{4} \times \frac{2}{5} [/tex]

[tex] = \frac{6}{20} [/tex]

[tex] = \frac{3}{10} [/tex]

That's it.

What are the coordinates for the park ?

Answers

Answer: hey panini

Step-by-step explanation:

3
Describe and correct the error a student made when graphing the linear equation y = -3/4x -6

Answers

Answer:

In step 1 the y-intercept should be plotted at (0,-6)

Step-by-step explanation:

Remember that to find the Y intercept in any linear equation you need to use 0 as your X value, this means taking the formula in the y=mx+b form and replacing X with a 0.

Since the formula is y = -3/4x -6

We just insert a 0 insted of the "x"

y = -3/4(0) -6

y=0-6

y=6

So the y-intercept sould be placed in (0,-6)

That's what he did wrong when graphing the equation.

Answer:

Remember that to find the Y intercept in any linear equation you need to use 0 as your X value, this means taking the formula in the y=mx+b form and replacing X with a 0.

Step-by-step explanation:

Remember that to find the Y intercept in any linear equation you need to use 0 as your X value, this means taking the formula in the y=mx+b form and replacing X with a 0.

Since the formula is y = -3/4x -6

We just insert a 0 insted of the "x"

y = -3/4(0) -6

y=0-6

y=6

So the y-intercept sould be placed in (0,-6)

That's what he did wrong when graphing the equation.

A composition of transformations maps pre-image EFGH to final image E"F"G"H".

Trapezoid E F G H is shifted down and to the left to trapezoid E prime F prime G prime H prime. Trapezoid E prime F prime G prime H prime is reflected across the line of reflection c to form trapezoid E double-prime F double-prime G double-prime H double-prime.

The first transformation for this composition is [________], and the second transformation is a reflection across line c.

a 180° rotation about point G
a 180° rotation about point H
a translation down and to the left
a translation up and to the right

Answers

Answer:

c. a translation down and to the left

Step-by-step explanation:

As per translation of a graph, the first transformation for this composition is translation down and to the left.

What is the translation of a graph?

"A translation of a graph is its rigid movement, vertically or horizontally."

Here, A composition of transformations maps pre-image EFGH to final image E"F"G"H".

Trapezoid E F G H is shifted down and to the left to trapezoid E prime F prime G prime H prime.

Therefore, this transformation for this composition is translation down and to the left, and the second transformation is a reflection across line c.

Learn more about translation of a graph here: https://brainly.com/question/11788591

#SPJ3

is this correct I made an educated guess because I don't really understand it

Answers

Answer:

Option (2)

Step-by-step explanation:

A function 'f' has been given as,

f(x) = 3x + 1

And the inverse of this function is,

[tex]f^{-1}(x)=\frac{x-1}{3}[/tex]

We have to find the value of f(2).

So we will use the expression: f(x) = 3x + 1

By substituting x = 2 in this function,

f(2) = 3×2 + 1

     = 6 + 1

     = 7

This result tells us the value of the function at x = 2.

Therefore, Option (2) will be the answer.

3.2654376... Rational or Irrational *
O Rational
O Irrational

Answers

Answer:

its irrational if the number goes on forever my g

Yes it’s irrational because you can’t predict what the next number is.

Write a linear equation that passes through the points (1,-6) and (-3, 10) *


HAHAHA HELP PLS

Answers

y=-4x-2

Step-by-step explanation:

Answer:

y= -4x-2

Step-by-step explanation:

10- -6/-3-1

10+6/-3-1

16/-4

-4



Rewrite it as a function and isolate Y. ​

Answers

Answer: y= -2x + 12

or f (y ) = 6 - [tex]\frac{y}{2}[/tex]

Step-by-step explanation:

just subtract the 2x to get the y alone

y2 - 3y + 7 for y= -2

please help me

Answers

Answer:9

Step-by-step explanation: -2x2-3x-2+7=9

Answer:

9

Step-by-step explanation:

(-2x2) - (3x-2) + 7 = 9

Lucas owed his friend 12$. He got some money from babysitting and was able to pay his friend back $8. What is his balance now?
~find an equation to match this word problem and solve~
Plz i need it right now thanksss

Answers

Answer:

balance $4

Step-by-step explanation:

12 - 8 = 4

Answer:

$4

Step-by-step explanation:

$ = 12 - 8

12 - 8 = $4

The body mass index (BMI) of a person is the person’s weight divided by the square of his or her height. It is an indirect measure of the person’s body fat and an indicator of obesity. Results from surveys conducted by the Centers for Disease Control and Prevention (CDC) showed that the estimated mean BMI for US adults increased from 25.0 in the 1960–1962 period to 28.1 in the 1999–2002 period.

Suppose you are a health researcher. You conduct a hypothesis test to determine whether the mean BMI of US adults in the current year is lower than the mean BMI of US adults in 2000. Assume that the mean BMI of US adults in 2000 was 28.1 (the population mean). You obtain a sample of BMI measurements of 1,034 US adults, which yields a sample mean of M = 28.9. Let μ denote the mean BMI of US adults in the current year.

Required:
Formulate your null and alternative hypotheses by selecting the appropriate values in the blue dropdown menus that follow.

Answers

Answer:

The null hypothesis is  [tex]H_o : \mu = 28.1[/tex]

 The alternative hypothesis is  [tex]H_a : \mu < 28.1[/tex]

Step-by-step explanation:

From the question we are told that

   The population mean at 1960 - 1962 is  [tex]\mu_1 = 25.0[/tex]

    The population mean at 1999- 2002 is  [tex]\mu_2 = 28.1[/tex]

     The sample size is  n=  1034

     The sample mean is  [tex]M = 28.9[/tex]

 The null hypothesis is  [tex]H_o : \mu = 28.1[/tex]

 The alternative hypothesis is  [tex]H_a : \mu < 28.1[/tex]

Sammie bought just enough fencing to border either a rectangular plot or a square plot,

shown. The perimeters of the plots are the same. How many feet of fencing did she buy?


(3x + 6) feet.

———————

| |

| | (x - 1) feet

|__________|


(x + 6) feet

___________

| |

| |

| | (x + 6) feet

| |

|__________ |


Sammie bought____feet of fencing.

Answers

Answer:

3.5 feet

Step-by-step explanation:

The question isn't formatted properly.

We have:

(x + 6) feet on each side of the square

(3x + 6) feet horizontally on the rectangle

(x - 1) feet vertically on the rectangular

First, we need to determine the perimeter of the square:

[tex]Perimeter = 4 * Length[/tex]

[tex]Perimeter = 4 * (x + 6)[/tex]

[tex]Perimeter = 4x + 24[/tex]

Next, we determine the perimeter of the rectangle

[tex]Perimeter = 2(Length + Width)[/tex]

[tex]Perimeter = 2(3x + 6 + x - 1)[/tex]

[tex]Perimeter = 2(3x + x+ 6 - 1)[/tex]

[tex]Perimeter = 2(4x+ 5)[/tex]

[tex]Perimeter = 8x+ 10[/tex]

Equate both perimeters:

[tex]8x + 10 = 4x + 24[/tex]

Collect Like Terms

[tex]8x - 4x = 24 - 10[/tex]

[tex]4x = 14[/tex]

Solve for x

[tex]x = 14/4[/tex]

[tex]x = 3.5[/tex]

Kevin’s school is 5.2 miles away from his house. How far is this in yards? Do not round your answer. _________ yards

Answers

Answer:

9152 yd

Step-by-step explanation:

s x = –4 a valid solution this linear equation? 2x + 9(x – 1) = 8(2x + 2) – 5 No. When –4 is substituted for the variable, the result is a true statement. No. When –4 is substituted for the variable, the result is a false statement. Yes. When –4 is substituted for the variable, the result is a true statement. Yes. When –4 is substituted for the variable, the result is a false statement.

Answers

Answer:your answer is C!

Step-by-step explanation: I got it right

Answer:

c) Yes. When –4 is substituted for the variable, the result is a true statement.

Step-by-step explanation:

i verified with my teacher may i plz get brainliest

use benchmarks to estimate the sum 11/15 +1/8

Answers

Answer:

103/120

Step-by-step explanation:

What is the slope of the line whose equation is 12=4x-6y?

Answers

Answer:

slope is 2/3

Step-by-step explanation:

12=4x-6y

We add 6y both sides

12+6y=4x

than subtract 12 from both sides

6y=4x-12

than divide both sides by 6

y=2/3x-2

and the slope is  2/3

At a middle school, 3 out of 4 students ride a bus to school. There are 600 studenta at the school. How many students ride a bus to school

Answers

Answer:

450 students ride a bus to school.

Step-by-step explanation:

If 3 out of 4 middle school students attend class by bus, it could be stated that 75% of the students use this means of transport to get to class (3/4 = 0.75).

Therefore, if this school has a total of 600 students, and 75% of them use the bus, it could be stated that 450 students attend school using this means of transport (600 x 75/100 = 450).

Answer:

450 students

Step-by-step explanation:

1/4 = 0.25

3/4 = 0.75

600 x 0.75 = 450 students

Hope this helps!

The price of a sweater was reduced from $20 to $12, By what percentage was the price of the sweater reduced?

Answers

Answer:

40%

Step-by-step explanation:

12 ÷ 20 = .6

You can convert the .6 to a percentage by multiplying it:

.6 x 100 = 60%

If 12 is 60% of 20, then that means 40% was taken away.

 Price of the sweater was reduced by 40%.

   Given in the question,

Original Price of a sweater is $20.Price of the sweater was reduced from $20 to $12.

Percentage reduction in the price from $20 to $12 can be calculated from the expression,

Percentage reduction = [tex]\frac{\text{Change in price}}{\text{Original price}}\times 100[/tex]

                                     = [tex](\frac{20-12}{12})\times 100[/tex]

                                     = [tex]\frac{8}{12}\times 100[/tex]

                                     = 40%

     Therefore, price of the sweater was reduced by 40%.

Learn more,

https://brainly.com/question/877552

what is the solution to -3/4x + 2

Answers

Answer: 1/4x(3x+8)

Step-by-step explanation:

hope this helps

let me know if its correct!!

From the table below, determine whether the data shows an exponential function. Explain why or why not.
х
3 1 -1 -3
у
1 2 3 4
a. No; the domain values are at regular intervals and the range values have a common sum 1.
b. No; the domain values are not at regular intervals.
C. Yes; the domain values are at regular intervals and the range values have a common factor 2.
d. Yes; the domain values are at regular intervals and the range values have a common sum 1.

Answers

Answer:

B because the domains repeat themselves so it can not be a function

No; the domain values are not at regular intervals.

The correct option is b.

What is an exponential function?

Mathematical functions with exponents include exponential functions. f(x) = bˣ, where b > 0 and b 1, is a fundamental exponential function.

In an exponential function, the domain values are usually at regular intervals, and the range values have a common factor or a common sum.

In the given data, the domain values are not at regular intervals.

The difference between 3 and 1 is 2, and the difference between 1 and -1 is 2, but the difference between -1 and -3 is -2.

This indicates that the domain values are not at regular intervals, which means that the data cannot be modeled by an exponential function.

Additionally, the range values also do not have a common factor or a common sum. The difference between consecutive y-values is 1, which is not a constant factor or sum.

Therefore, the data cannot be modeled by an exponential function.

To learn more about exponential;

https://brainly.com/question/14344314

#SPJ7

A map has a scale of 2 in: 6 mi. If Clayton
and Centerville are 10 in apart on the map
then how far apart are the real cities?

Answers

Both cities are 30 miles apart in real

Robert can complete 10 inspections in 8 hours. Which represents the unit rate for the number of inspections he completes per hour?
0.8 of an inspection in 1 hour
4 inspections in 5 hours
1.25 inspections in 1 hour
5 inspections in 4 hours

Answers

Answer:

1.25 inspections in an hour

so it is c!

10/8= 1.25

Answer:

its c

Step-by-step explanation:

lol xd rawr (i hate myself)

If I was asked to think of a number and multiply it by 2, I could
write this algebraically as 2x.
Write the following algebraically, using x as your unknown.
"I think of a number, multiply it by 2, subtract 3 and multiply the result by 4.​

Answers

Answer:

4(2x-3)

Step-by-step explanation:

Answer:

4 x (2x-3)

I hope it helps

Pls mark as Brainliest

Other Questions
Segun el informe como de puede describer la manera en la que el cine y la television presentan a los sonambulos brainliest for correct answer please help Point A is located at (-5,2). It is translated up 5 and left 3. Please state the location of A'. . Benjamin Franklin was opposed to injustice. Which details from the text support this statement? a=3 and c=2 10a - 3 - 5c - 2c Using this map, it can be determined that Solve for k: 4k+5a3=102k Fill in the blank with the correct German form.Ich _____ das Buch.lesenleseliestlestv Some plants produce a _________________________ between the plasma membrane and the primary cell wall. 20 points & BrainliestWhy are rotations a rigid motion? Spanish Questions I need help. 1. Summarize the authors argument and supporting evidence. 2. Why was Portugal successful in Asia? Be sure to amplify the authors argument as you explain your answer. Rishi Manchanda comments about all of the compelling stories that he hears about patients just like Veronica. Do you know anyone who has a similar story to Veronica's and has fallen through the cracks of our health care system? Do you feel that they would benefit from an organization like HealthBegins? Why or why not? A muffin recipe uses 334 cups of flour for every 12 cup of milk. Which proportions could you use to solve for the cups of milk needed for 513 cups of flour? Which is NOT a goal promoted by the government through fiscal policy? A) price stability B) economic growthC)full employment D) money supply Una de las tareas de la filosofa de la religin es: * 10 puntos Comprender el origen de los ritos Comprender la relacin entre Dios y el hombre Analizar la construccin social del concepto de Dios vinculado con las nociones ticas de la disciplina antropolgica. Construir un concepto de Dios In rupert's land fishing and ship building were the main industries. True or false? In which interval is the median?A sample of 50 students was surveyed about their timespent in front of a screen (laptop, tablet, phone, etc.).Each interval contains the left endpoint but not theright endpoint.[4,5)[5,6)[6,7)[7,8)Hours of Screen Time per Day10BFrequency26810 12 20)Why is there no competition in a monopoly?A)There are no close substitutes.B)There are too many substitutes.C)Prices are set by the government.D)The government forbids competition. What is the length of line segment KJ?O 2/3 unitsO 32 unitsM 3 vO 3/3 unitsO 375 units *newThe process of wave erosion is a relativelyphenomenon on Earth.10 pointsTrueFalse